Using Laplace Transforms to Solve DE

Click For Summary
The discussion revolves around solving the differential equation y'' - 4y = e^{-t} using Laplace transforms, with initial conditions y(0) = 1 and y'(0) = 0. The user attempts to apply the Laplace transform formulas but encounters difficulties in obtaining the correct solution, specifically in matching the expected answer of e^(2t). They express frustration over potential mistakes in their calculations, particularly in the transformation of the left side of the equation. The user seeks clarification and assistance in identifying errors in their approach. The thread highlights the common challenges faced when applying Laplace transforms to solve differential equations.
Ithryndil
Messages
142
Reaction score
0

Homework Statement


The problem is this:

y'' - 4y = e^{-t} , y(0) = 1, y'(0) = 0


Homework Equations


L{y(t)} = Y(s)
L{y'(t)} = sY(s) - y(0)
L{y''(t)} = s^2Y(s) - sy(0) - y(0)


The Attempt at a Solution



Ok, so I plugged the Laplace transforms for y'' and y into the equation as well as for e^(-t) and got:

Y(s) = \frac{1}{s(s-4)(s+1)} - \frac{1}{(s-4)} - \frac{4}{4(s+1)}

From that point on I would need to perform a partial fraction decomposition on the first term and last term, the middle term is ok. Doing so I get:

Y(s) = \frac{-1}{4s} + \frac{-1}{20(s-4)} + \frac{1}{5(s+1)}- \frac{1}{(s-1)} + \frac{1}{s} - \frac{1}{(s-4)}

However, the book is getting answers with e^(2t). I am not seeing how anything I have in that last line will yield that, so I assume I am doing something wrong.
 
Physics news on Phys.org
Check over the transform of the left side. I believe you are supposed to get

s2Y(s)-sY(0)-Y'(0)-4Y(s)
 
...It's stupid mistakes like the one I made above that are frustrating. Thank you.
 
Question: A clock's minute hand has length 4 and its hour hand has length 3. What is the distance between the tips at the moment when it is increasing most rapidly?(Putnam Exam Question) Answer: Making assumption that both the hands moves at constant angular velocities, the answer is ## \sqrt{7} .## But don't you think this assumption is somewhat doubtful and wrong?

Similar threads

  • · Replies 7 ·
Replies
7
Views
2K
  • · Replies 1 ·
Replies
1
Views
1K
  • · Replies 8 ·
Replies
8
Views
2K
Replies
2
Views
2K
  • · Replies 1 ·
Replies
1
Views
1K
  • · Replies 4 ·
Replies
4
Views
3K
Replies
9
Views
2K
  • · Replies 1 ·
Replies
1
Views
1K
  • · Replies 6 ·
Replies
6
Views
2K
Replies
3
Views
2K